Matemáticas, pregunta formulada por kevinzzz23, hace 17 horas

Desarrolla el teorema de De-Moivre en la siguiente raíz.

 \sqrt[4]{3 - 3i}

Respuestas a la pregunta

Contestado por deatttre
0

Respuesta:

QUE M***** ES ESO JAJAJAJA

Contestado por naulaallaicaleonel
0

Respuesta:

es verdad es verdadero

Yyjehdh eyeyhehe dhhdhfhfhf jsjsbgauq djje


deatttre: buen trabajo JAJA
deatttre: para que estudie
Otras preguntas